A sequence $a_n$ is such that $lim_n(a_n+1 - a_n) = 0$. Given some additional properties of $a_n$ prove that it converges. Announcing the arrival of Valued Associate #679: Cesar Manara Planned maintenance scheduled April 17/18, 2019 at 00:00UTC (8:00pm US/Eastern)If $sum a_n$ converges and $b_n=sumlimits_k=n^inftya_n $, prove that $sum fraca_nb_n$ divergesTo show sequence $a_n$ is convergentSeries convergence proof reviewProb. 8, Chap. 3 in Baby Rudin: If $sum a_n$ converges and $leftb_nright$ is monotonic and bounded, then $sum a_n b_n$ converges.Prove that the alternating series converges.Given that $sumlimits_n=1^inftya_n$ converges ($a_n >0$), then does $sumlimits_n=1^inftya_n^3 sin(n)$ converge?Prove that $ lim_n fraca_1 + cdots + a_nn=L$$maxa_1,a_2,dots,a_n$ converges for a convergent sequence $a_n$Let $sum_k=1^infty a_n$ be convergent show that $sum_k=1^infty n(a_n-a_n+1)$ convergesShow that $(a_n)_ninmathbbN$ converges, given that $|a_n-a_n+1|leqlambda |a_n-1-a_n| $

What causes the vertical darker bands in my photo?

How do I mention the quality of my school without bragging

Sorting numerically

G-Code for resetting to 100% speed

Is above average number of years spent on PhD considered a red flag in future academia or industry positions?

What is the longest distance a 13th-level monk can jump while attacking on the same turn?

Is there a service that would inform me whenever a new direct route is scheduled from a given airport?

Did Kevin spill real chili?

How widely used is the term Treppenwitz? Is it something that most Germans know?

How to draw this diagram using TikZ package?

How does cp -a work

Antler Helmet: Can it work?

How do I keep my slimes from escaping their pens?

Do I really need recursive chmod to restrict access to a folder?

Does accepting a pardon have any bearing on trying that person for the same crime in a sovereign jurisdiction?

Are my PIs rude or am I just being too sensitive?

How can I make names more distinctive without making them longer?

What does the "x" in "x86" represent?

How do I stop a creek from eroding my steep embankment?

"Seemed to had" is it correct?

How to assign captions for two tables in LaTeX?

Gastric acid as a weapon

What are the motives behind Cersei's orders given to Bronn?

Should gear shift center itself while in neutral?



A sequence $a_n$ is such that $lim_n(a_n+1 - a_n) = 0$. Given some additional properties of $a_n$ prove that it converges.



Announcing the arrival of Valued Associate #679: Cesar Manara
Planned maintenance scheduled April 17/18, 2019 at 00:00UTC (8:00pm US/Eastern)If $sum a_n$ converges and $b_n=sumlimits_k=n^inftya_n $, prove that $sum fraca_nb_n$ divergesTo show sequence $a_n$ is convergentSeries convergence proof reviewProb. 8, Chap. 3 in Baby Rudin: If $sum a_n$ converges and $leftb_nright$ is monotonic and bounded, then $sum a_n b_n$ converges.Prove that the alternating series converges.Given that $sumlimits_n=1^inftya_n$ converges ($a_n >0$), then does $sumlimits_n=1^inftya_n^3 sin(n)$ converge?Prove that $ lim_n fraca_1 + cdots + a_nn=L$$maxa_1,a_2,dots,a_n$ converges for a convergent sequence $a_n$Let $sum_k=1^infty a_n$ be convergent show that $sum_k=1^infty n(a_n-a_n+1)$ convergesShow that $(a_n)_ninmathbbN$ converges, given that $|a_n-a_n+1|leqlambda |a_n-1-a_n| $










1












$begingroup$



Let $a_n, ninBbb N$ be a sequence such that:
$$
lim_ntoinfty(a_n+1 - a_n) = 0tag1
$$

The sequence also satisfies the following properties:
$$
beginalign*
|a_n+2-a_n+1| le |a_n+1-a_n|tag 2\
(a_n+2 - a_n+1)(a_n+1-a_n) le 0 tag 3
endalign*
$$

Prove $a_n$ converges.




While looking at the problem first time I was thinking that $(1)$ implies convergnce of $a_n$, but looking more carefully one may think of a harmonic series:
$$
a_n = sum_k=1^n1over k
$$

Which is clearly divergent as $ntoinfty$, even though $(1)$ still holds. So obviously there is some extra work to do. So below are some of my further findings. First, let's define a new sequence:
$$
b_n = a_n - a_n-1tag 4
$$

If a sequence converges then its absolute values also converge:
$$
lim_ntoinftyb_n = 0 implies lim_ntoinfty|b_n| = 0
$$

If we now take a look at $(3)$ there are two possible cases for that statement to hold, either:
$$
b_n+2 ge 0\
b_n+1 le 0 tag5
$$

or:
$$
b_n+2 le 0\
b_n+1 ge 0 tag6
$$

But at the same time:
$$
b_n+2b_n+1 le 0\
b_n+1b_n le 0\
b_nb_n-1 le 0\
cdots
$$

Let's stick to case $(5)$ for definiteness. This implies:
$$
0 le b_n+2le b_n le b_n-2 le cdots tag7
$$

At the same time since $|b_n|$ is monotically decreasing and $b_n+1le 0$:
$$
cdots le b_n-3 le b_n-1 le b_n+1 cdots le 0 tag8
$$



It looks like $b_n$ is an alternating sequence, which consists of two subsequences both convergent to $0$, that is because $(7)$ is monotonically decreasing towards $0$ and $(8)$ is monotonically increasing towards $0$. And this depends on whether we assume $(5)$ or $(6)$.



That is where I got stuck. I do not see how to combine those finding to show that $a_n$ is actually convergent. How do I proceed?










share|cite|improve this question











$endgroup$







  • 1




    $begingroup$
    $|b_n|$ is convergent because $b_n$ is convergent, you overcomplicated your proof of $b_n$'s convergence :)
    $endgroup$
    – 5xum
    Mar 26 at 7:51










  • $begingroup$
    @YiFan This is how it's stated in the book. Consider $a_n = sum 1over k$, then $lim_n (a_n+1 - a_n) = lim_n 1over n+1 = 0$, but $lim_n a_n = lim_n sum 1over k = +infty$
    $endgroup$
    – roman
    Mar 26 at 8:12















1












$begingroup$



Let $a_n, ninBbb N$ be a sequence such that:
$$
lim_ntoinfty(a_n+1 - a_n) = 0tag1
$$

The sequence also satisfies the following properties:
$$
beginalign*
|a_n+2-a_n+1| le |a_n+1-a_n|tag 2\
(a_n+2 - a_n+1)(a_n+1-a_n) le 0 tag 3
endalign*
$$

Prove $a_n$ converges.




While looking at the problem first time I was thinking that $(1)$ implies convergnce of $a_n$, but looking more carefully one may think of a harmonic series:
$$
a_n = sum_k=1^n1over k
$$

Which is clearly divergent as $ntoinfty$, even though $(1)$ still holds. So obviously there is some extra work to do. So below are some of my further findings. First, let's define a new sequence:
$$
b_n = a_n - a_n-1tag 4
$$

If a sequence converges then its absolute values also converge:
$$
lim_ntoinftyb_n = 0 implies lim_ntoinfty|b_n| = 0
$$

If we now take a look at $(3)$ there are two possible cases for that statement to hold, either:
$$
b_n+2 ge 0\
b_n+1 le 0 tag5
$$

or:
$$
b_n+2 le 0\
b_n+1 ge 0 tag6
$$

But at the same time:
$$
b_n+2b_n+1 le 0\
b_n+1b_n le 0\
b_nb_n-1 le 0\
cdots
$$

Let's stick to case $(5)$ for definiteness. This implies:
$$
0 le b_n+2le b_n le b_n-2 le cdots tag7
$$

At the same time since $|b_n|$ is monotically decreasing and $b_n+1le 0$:
$$
cdots le b_n-3 le b_n-1 le b_n+1 cdots le 0 tag8
$$



It looks like $b_n$ is an alternating sequence, which consists of two subsequences both convergent to $0$, that is because $(7)$ is monotonically decreasing towards $0$ and $(8)$ is monotonically increasing towards $0$. And this depends on whether we assume $(5)$ or $(6)$.



That is where I got stuck. I do not see how to combine those finding to show that $a_n$ is actually convergent. How do I proceed?










share|cite|improve this question











$endgroup$







  • 1




    $begingroup$
    $|b_n|$ is convergent because $b_n$ is convergent, you overcomplicated your proof of $b_n$'s convergence :)
    $endgroup$
    – 5xum
    Mar 26 at 7:51










  • $begingroup$
    @YiFan This is how it's stated in the book. Consider $a_n = sum 1over k$, then $lim_n (a_n+1 - a_n) = lim_n 1over n+1 = 0$, but $lim_n a_n = lim_n sum 1over k = +infty$
    $endgroup$
    – roman
    Mar 26 at 8:12













1












1








1





$begingroup$



Let $a_n, ninBbb N$ be a sequence such that:
$$
lim_ntoinfty(a_n+1 - a_n) = 0tag1
$$

The sequence also satisfies the following properties:
$$
beginalign*
|a_n+2-a_n+1| le |a_n+1-a_n|tag 2\
(a_n+2 - a_n+1)(a_n+1-a_n) le 0 tag 3
endalign*
$$

Prove $a_n$ converges.




While looking at the problem first time I was thinking that $(1)$ implies convergnce of $a_n$, but looking more carefully one may think of a harmonic series:
$$
a_n = sum_k=1^n1over k
$$

Which is clearly divergent as $ntoinfty$, even though $(1)$ still holds. So obviously there is some extra work to do. So below are some of my further findings. First, let's define a new sequence:
$$
b_n = a_n - a_n-1tag 4
$$

If a sequence converges then its absolute values also converge:
$$
lim_ntoinftyb_n = 0 implies lim_ntoinfty|b_n| = 0
$$

If we now take a look at $(3)$ there are two possible cases for that statement to hold, either:
$$
b_n+2 ge 0\
b_n+1 le 0 tag5
$$

or:
$$
b_n+2 le 0\
b_n+1 ge 0 tag6
$$

But at the same time:
$$
b_n+2b_n+1 le 0\
b_n+1b_n le 0\
b_nb_n-1 le 0\
cdots
$$

Let's stick to case $(5)$ for definiteness. This implies:
$$
0 le b_n+2le b_n le b_n-2 le cdots tag7
$$

At the same time since $|b_n|$ is monotically decreasing and $b_n+1le 0$:
$$
cdots le b_n-3 le b_n-1 le b_n+1 cdots le 0 tag8
$$



It looks like $b_n$ is an alternating sequence, which consists of two subsequences both convergent to $0$, that is because $(7)$ is monotonically decreasing towards $0$ and $(8)$ is monotonically increasing towards $0$. And this depends on whether we assume $(5)$ or $(6)$.



That is where I got stuck. I do not see how to combine those finding to show that $a_n$ is actually convergent. How do I proceed?










share|cite|improve this question











$endgroup$





Let $a_n, ninBbb N$ be a sequence such that:
$$
lim_ntoinfty(a_n+1 - a_n) = 0tag1
$$

The sequence also satisfies the following properties:
$$
beginalign*
|a_n+2-a_n+1| le |a_n+1-a_n|tag 2\
(a_n+2 - a_n+1)(a_n+1-a_n) le 0 tag 3
endalign*
$$

Prove $a_n$ converges.




While looking at the problem first time I was thinking that $(1)$ implies convergnce of $a_n$, but looking more carefully one may think of a harmonic series:
$$
a_n = sum_k=1^n1over k
$$

Which is clearly divergent as $ntoinfty$, even though $(1)$ still holds. So obviously there is some extra work to do. So below are some of my further findings. First, let's define a new sequence:
$$
b_n = a_n - a_n-1tag 4
$$

If a sequence converges then its absolute values also converge:
$$
lim_ntoinftyb_n = 0 implies lim_ntoinfty|b_n| = 0
$$

If we now take a look at $(3)$ there are two possible cases for that statement to hold, either:
$$
b_n+2 ge 0\
b_n+1 le 0 tag5
$$

or:
$$
b_n+2 le 0\
b_n+1 ge 0 tag6
$$

But at the same time:
$$
b_n+2b_n+1 le 0\
b_n+1b_n le 0\
b_nb_n-1 le 0\
cdots
$$

Let's stick to case $(5)$ for definiteness. This implies:
$$
0 le b_n+2le b_n le b_n-2 le cdots tag7
$$

At the same time since $|b_n|$ is monotically decreasing and $b_n+1le 0$:
$$
cdots le b_n-3 le b_n-1 le b_n+1 cdots le 0 tag8
$$



It looks like $b_n$ is an alternating sequence, which consists of two subsequences both convergent to $0$, that is because $(7)$ is monotonically decreasing towards $0$ and $(8)$ is monotonically increasing towards $0$. And this depends on whether we assume $(5)$ or $(6)$.



That is where I got stuck. I do not see how to combine those finding to show that $a_n$ is actually convergent. How do I proceed?







real-analysis sequences-and-series limits






share|cite|improve this question















share|cite|improve this question













share|cite|improve this question




share|cite|improve this question








edited Mar 26 at 7:55







roman

















asked Mar 26 at 7:47









romanroman

2,50221226




2,50221226







  • 1




    $begingroup$
    $|b_n|$ is convergent because $b_n$ is convergent, you overcomplicated your proof of $b_n$'s convergence :)
    $endgroup$
    – 5xum
    Mar 26 at 7:51










  • $begingroup$
    @YiFan This is how it's stated in the book. Consider $a_n = sum 1over k$, then $lim_n (a_n+1 - a_n) = lim_n 1over n+1 = 0$, but $lim_n a_n = lim_n sum 1over k = +infty$
    $endgroup$
    – roman
    Mar 26 at 8:12












  • 1




    $begingroup$
    $|b_n|$ is convergent because $b_n$ is convergent, you overcomplicated your proof of $b_n$'s convergence :)
    $endgroup$
    – 5xum
    Mar 26 at 7:51










  • $begingroup$
    @YiFan This is how it's stated in the book. Consider $a_n = sum 1over k$, then $lim_n (a_n+1 - a_n) = lim_n 1over n+1 = 0$, but $lim_n a_n = lim_n sum 1over k = +infty$
    $endgroup$
    – roman
    Mar 26 at 8:12







1




1




$begingroup$
$|b_n|$ is convergent because $b_n$ is convergent, you overcomplicated your proof of $b_n$'s convergence :)
$endgroup$
– 5xum
Mar 26 at 7:51




$begingroup$
$|b_n|$ is convergent because $b_n$ is convergent, you overcomplicated your proof of $b_n$'s convergence :)
$endgroup$
– 5xum
Mar 26 at 7:51












$begingroup$
@YiFan This is how it's stated in the book. Consider $a_n = sum 1over k$, then $lim_n (a_n+1 - a_n) = lim_n 1over n+1 = 0$, but $lim_n a_n = lim_n sum 1over k = +infty$
$endgroup$
– roman
Mar 26 at 8:12




$begingroup$
@YiFan This is how it's stated in the book. Consider $a_n = sum 1over k$, then $lim_n (a_n+1 - a_n) = lim_n 1over n+1 = 0$, but $lim_n a_n = lim_n sum 1over k = +infty$
$endgroup$
– roman
Mar 26 at 8:12










2 Answers
2






active

oldest

votes


















1












$begingroup$

It is convergence of $sum b_n$ (not that of $(b_n)$) that gives convergence of $(a_n)$. Apply Alternating Series Test to see that the series $sum b_n$ is convergent. Then note that $b_1+b_2+...+b_n=a_1-a_n$. Hence $lim a_n$ exists.






share|cite|improve this answer











$endgroup$




















    1












    $begingroup$

    Ok, following @Kavi Rama Murthy's suggestions here is what I eventually got (hopefully correct).



    I'm still going to stick to how $b_n$ is defined:
    $$
    lim_ntoinftyb_n = 0\
    |b_n+2| le |b_n+1|\
    b_n+2b_n+1 le 0
    $$

    Let's now define a partial sum $S_n$:
    $$
    S_n = sum_k=1^n (-1)^k-1|b_k|
    $$

    Here the $(-1)^k-1$ actually depends on which terms are positive and which are negative (which is defined by eiher $(5)$ or $(6)$ in the OP). Consider two partial sums for odd and even numbers of terms:
    $$
    beginalign*
    S_2p+1 = sum_k=1^2p+1(-1)^k-1|b_k| \
    S_2p = sum_k=1^2p(-1)^k-1|b_k|
    endalign*
    $$



    Consider the sum of odd amount of terms:
    $$
    beginalign
    S_2(p+1)+1 = &sum_k=1^2p+3(-1)^k-1|b_k| \
    &=sum_k=1^2p+1(-1)^k-1|b_k| underbrace_le0\
    &le S_2p+1
    endalign
    $$

    Which means $S_2p+1$ is monotonically decreasing.



    Now use the same approach for even number of terms:
    $$
    beginalign
    S_2(p+1) = &sum_k=1^2p+2(-1)^k-1|b_k| \
    &=sum_k=1^2p(-1)^k-1|b_k| + underbraceb_2p+2_ge0\
    &ge S_2p
    endalign
    $$

    Which means $S_2p$ is monotonically increasing. Consider the following:
    $$
    S_2p+1 - S_2p = |b_2p+1| ge 0tag*
    $$



    Now $S_2 = |b_1| - |b_2|$ and by monotoninity of $S_2p$:
    $$
    S_2 = |b_1| - |b_2| le S_2p
    $$



    At the same time $S_1 = |b_1|$ which again by monotonicity of $S_2p+1$:
    $$
    S_2p+1 le S_1 = |b_1|
    $$

    Now using (*):
    $$
    |b_1| - |b_2| le S_2p le S_2p+1 le |b_1|\
    S_2p-1 - S_2p = |b_2p+1| = |a_2p+1 - a_2p|
    $$

    But $|a_2p+1 - a_2p|$ is convergent as far as $(a_2p+1 - a_2p)$ is. Which yields:
    $$
    lim_ptoinfty(S_2p+1 - S_2p) = lim_ptoinfty|b_2p+1| = lim_ptoinfty|a_2p+1 - a_2p| = 0
    $$



    Finally:
    $$
    exists lim_ntoinftyS_n implies exists lim_ntoinftysum_k=1^n b_k = L
    $$

    Thus:
    $$
    lim_ntoinftysum_k=1^n b_k = lim_ntoinfty(a_n - a_1) = L \
    implies lim_ntoinftya_n = L + a_1
    $$



    Thus $a_n$ is convergent.






    share|cite|improve this answer









    $endgroup$













      Your Answer








      StackExchange.ready(function()
      var channelOptions =
      tags: "".split(" "),
      id: "69"
      ;
      initTagRenderer("".split(" "), "".split(" "), channelOptions);

      StackExchange.using("externalEditor", function()
      // Have to fire editor after snippets, if snippets enabled
      if (StackExchange.settings.snippets.snippetsEnabled)
      StackExchange.using("snippets", function()
      createEditor();
      );

      else
      createEditor();

      );

      function createEditor()
      StackExchange.prepareEditor(
      heartbeatType: 'answer',
      autoActivateHeartbeat: false,
      convertImagesToLinks: true,
      noModals: true,
      showLowRepImageUploadWarning: true,
      reputationToPostImages: 10,
      bindNavPrevention: true,
      postfix: "",
      imageUploader:
      brandingHtml: "Powered by u003ca class="icon-imgur-white" href="https://imgur.com/"u003eu003c/au003e",
      contentPolicyHtml: "User contributions licensed under u003ca href="https://creativecommons.org/licenses/by-sa/3.0/"u003ecc by-sa 3.0 with attribution requiredu003c/au003e u003ca href="https://stackoverflow.com/legal/content-policy"u003e(content policy)u003c/au003e",
      allowUrls: true
      ,
      noCode: true, onDemand: true,
      discardSelector: ".discard-answer"
      ,immediatelyShowMarkdownHelp:true
      );



      );













      draft saved

      draft discarded


















      StackExchange.ready(
      function ()
      StackExchange.openid.initPostLogin('.new-post-login', 'https%3a%2f%2fmath.stackexchange.com%2fquestions%2f3162857%2fa-sequence-a-n-is-such-that-lim-na-n1-a-n-0-given-some-additi%23new-answer', 'question_page');

      );

      Post as a guest















      Required, but never shown

























      2 Answers
      2






      active

      oldest

      votes








      2 Answers
      2






      active

      oldest

      votes









      active

      oldest

      votes






      active

      oldest

      votes









      1












      $begingroup$

      It is convergence of $sum b_n$ (not that of $(b_n)$) that gives convergence of $(a_n)$. Apply Alternating Series Test to see that the series $sum b_n$ is convergent. Then note that $b_1+b_2+...+b_n=a_1-a_n$. Hence $lim a_n$ exists.






      share|cite|improve this answer











      $endgroup$

















        1












        $begingroup$

        It is convergence of $sum b_n$ (not that of $(b_n)$) that gives convergence of $(a_n)$. Apply Alternating Series Test to see that the series $sum b_n$ is convergent. Then note that $b_1+b_2+...+b_n=a_1-a_n$. Hence $lim a_n$ exists.






        share|cite|improve this answer











        $endgroup$















          1












          1








          1





          $begingroup$

          It is convergence of $sum b_n$ (not that of $(b_n)$) that gives convergence of $(a_n)$. Apply Alternating Series Test to see that the series $sum b_n$ is convergent. Then note that $b_1+b_2+...+b_n=a_1-a_n$. Hence $lim a_n$ exists.






          share|cite|improve this answer











          $endgroup$



          It is convergence of $sum b_n$ (not that of $(b_n)$) that gives convergence of $(a_n)$. Apply Alternating Series Test to see that the series $sum b_n$ is convergent. Then note that $b_1+b_2+...+b_n=a_1-a_n$. Hence $lim a_n$ exists.







          share|cite|improve this answer














          share|cite|improve this answer



          share|cite|improve this answer








          edited Mar 26 at 8:48

























          answered Mar 26 at 7:51









          Kavi Rama MurthyKavi Rama Murthy

          74.9k53270




          74.9k53270





















              1












              $begingroup$

              Ok, following @Kavi Rama Murthy's suggestions here is what I eventually got (hopefully correct).



              I'm still going to stick to how $b_n$ is defined:
              $$
              lim_ntoinftyb_n = 0\
              |b_n+2| le |b_n+1|\
              b_n+2b_n+1 le 0
              $$

              Let's now define a partial sum $S_n$:
              $$
              S_n = sum_k=1^n (-1)^k-1|b_k|
              $$

              Here the $(-1)^k-1$ actually depends on which terms are positive and which are negative (which is defined by eiher $(5)$ or $(6)$ in the OP). Consider two partial sums for odd and even numbers of terms:
              $$
              beginalign*
              S_2p+1 = sum_k=1^2p+1(-1)^k-1|b_k| \
              S_2p = sum_k=1^2p(-1)^k-1|b_k|
              endalign*
              $$



              Consider the sum of odd amount of terms:
              $$
              beginalign
              S_2(p+1)+1 = &sum_k=1^2p+3(-1)^k-1|b_k| \
              &=sum_k=1^2p+1(-1)^k-1|b_k| underbrace_le0\
              &le S_2p+1
              endalign
              $$

              Which means $S_2p+1$ is monotonically decreasing.



              Now use the same approach for even number of terms:
              $$
              beginalign
              S_2(p+1) = &sum_k=1^2p+2(-1)^k-1|b_k| \
              &=sum_k=1^2p(-1)^k-1|b_k| + underbraceb_2p+2_ge0\
              &ge S_2p
              endalign
              $$

              Which means $S_2p$ is monotonically increasing. Consider the following:
              $$
              S_2p+1 - S_2p = |b_2p+1| ge 0tag*
              $$



              Now $S_2 = |b_1| - |b_2|$ and by monotoninity of $S_2p$:
              $$
              S_2 = |b_1| - |b_2| le S_2p
              $$



              At the same time $S_1 = |b_1|$ which again by monotonicity of $S_2p+1$:
              $$
              S_2p+1 le S_1 = |b_1|
              $$

              Now using (*):
              $$
              |b_1| - |b_2| le S_2p le S_2p+1 le |b_1|\
              S_2p-1 - S_2p = |b_2p+1| = |a_2p+1 - a_2p|
              $$

              But $|a_2p+1 - a_2p|$ is convergent as far as $(a_2p+1 - a_2p)$ is. Which yields:
              $$
              lim_ptoinfty(S_2p+1 - S_2p) = lim_ptoinfty|b_2p+1| = lim_ptoinfty|a_2p+1 - a_2p| = 0
              $$



              Finally:
              $$
              exists lim_ntoinftyS_n implies exists lim_ntoinftysum_k=1^n b_k = L
              $$

              Thus:
              $$
              lim_ntoinftysum_k=1^n b_k = lim_ntoinfty(a_n - a_1) = L \
              implies lim_ntoinftya_n = L + a_1
              $$



              Thus $a_n$ is convergent.






              share|cite|improve this answer









              $endgroup$

















                1












                $begingroup$

                Ok, following @Kavi Rama Murthy's suggestions here is what I eventually got (hopefully correct).



                I'm still going to stick to how $b_n$ is defined:
                $$
                lim_ntoinftyb_n = 0\
                |b_n+2| le |b_n+1|\
                b_n+2b_n+1 le 0
                $$

                Let's now define a partial sum $S_n$:
                $$
                S_n = sum_k=1^n (-1)^k-1|b_k|
                $$

                Here the $(-1)^k-1$ actually depends on which terms are positive and which are negative (which is defined by eiher $(5)$ or $(6)$ in the OP). Consider two partial sums for odd and even numbers of terms:
                $$
                beginalign*
                S_2p+1 = sum_k=1^2p+1(-1)^k-1|b_k| \
                S_2p = sum_k=1^2p(-1)^k-1|b_k|
                endalign*
                $$



                Consider the sum of odd amount of terms:
                $$
                beginalign
                S_2(p+1)+1 = &sum_k=1^2p+3(-1)^k-1|b_k| \
                &=sum_k=1^2p+1(-1)^k-1|b_k| underbrace_le0\
                &le S_2p+1
                endalign
                $$

                Which means $S_2p+1$ is monotonically decreasing.



                Now use the same approach for even number of terms:
                $$
                beginalign
                S_2(p+1) = &sum_k=1^2p+2(-1)^k-1|b_k| \
                &=sum_k=1^2p(-1)^k-1|b_k| + underbraceb_2p+2_ge0\
                &ge S_2p
                endalign
                $$

                Which means $S_2p$ is monotonically increasing. Consider the following:
                $$
                S_2p+1 - S_2p = |b_2p+1| ge 0tag*
                $$



                Now $S_2 = |b_1| - |b_2|$ and by monotoninity of $S_2p$:
                $$
                S_2 = |b_1| - |b_2| le S_2p
                $$



                At the same time $S_1 = |b_1|$ which again by monotonicity of $S_2p+1$:
                $$
                S_2p+1 le S_1 = |b_1|
                $$

                Now using (*):
                $$
                |b_1| - |b_2| le S_2p le S_2p+1 le |b_1|\
                S_2p-1 - S_2p = |b_2p+1| = |a_2p+1 - a_2p|
                $$

                But $|a_2p+1 - a_2p|$ is convergent as far as $(a_2p+1 - a_2p)$ is. Which yields:
                $$
                lim_ptoinfty(S_2p+1 - S_2p) = lim_ptoinfty|b_2p+1| = lim_ptoinfty|a_2p+1 - a_2p| = 0
                $$



                Finally:
                $$
                exists lim_ntoinftyS_n implies exists lim_ntoinftysum_k=1^n b_k = L
                $$

                Thus:
                $$
                lim_ntoinftysum_k=1^n b_k = lim_ntoinfty(a_n - a_1) = L \
                implies lim_ntoinftya_n = L + a_1
                $$



                Thus $a_n$ is convergent.






                share|cite|improve this answer









                $endgroup$















                  1












                  1








                  1





                  $begingroup$

                  Ok, following @Kavi Rama Murthy's suggestions here is what I eventually got (hopefully correct).



                  I'm still going to stick to how $b_n$ is defined:
                  $$
                  lim_ntoinftyb_n = 0\
                  |b_n+2| le |b_n+1|\
                  b_n+2b_n+1 le 0
                  $$

                  Let's now define a partial sum $S_n$:
                  $$
                  S_n = sum_k=1^n (-1)^k-1|b_k|
                  $$

                  Here the $(-1)^k-1$ actually depends on which terms are positive and which are negative (which is defined by eiher $(5)$ or $(6)$ in the OP). Consider two partial sums for odd and even numbers of terms:
                  $$
                  beginalign*
                  S_2p+1 = sum_k=1^2p+1(-1)^k-1|b_k| \
                  S_2p = sum_k=1^2p(-1)^k-1|b_k|
                  endalign*
                  $$



                  Consider the sum of odd amount of terms:
                  $$
                  beginalign
                  S_2(p+1)+1 = &sum_k=1^2p+3(-1)^k-1|b_k| \
                  &=sum_k=1^2p+1(-1)^k-1|b_k| underbrace_le0\
                  &le S_2p+1
                  endalign
                  $$

                  Which means $S_2p+1$ is monotonically decreasing.



                  Now use the same approach for even number of terms:
                  $$
                  beginalign
                  S_2(p+1) = &sum_k=1^2p+2(-1)^k-1|b_k| \
                  &=sum_k=1^2p(-1)^k-1|b_k| + underbraceb_2p+2_ge0\
                  &ge S_2p
                  endalign
                  $$

                  Which means $S_2p$ is monotonically increasing. Consider the following:
                  $$
                  S_2p+1 - S_2p = |b_2p+1| ge 0tag*
                  $$



                  Now $S_2 = |b_1| - |b_2|$ and by monotoninity of $S_2p$:
                  $$
                  S_2 = |b_1| - |b_2| le S_2p
                  $$



                  At the same time $S_1 = |b_1|$ which again by monotonicity of $S_2p+1$:
                  $$
                  S_2p+1 le S_1 = |b_1|
                  $$

                  Now using (*):
                  $$
                  |b_1| - |b_2| le S_2p le S_2p+1 le |b_1|\
                  S_2p-1 - S_2p = |b_2p+1| = |a_2p+1 - a_2p|
                  $$

                  But $|a_2p+1 - a_2p|$ is convergent as far as $(a_2p+1 - a_2p)$ is. Which yields:
                  $$
                  lim_ptoinfty(S_2p+1 - S_2p) = lim_ptoinfty|b_2p+1| = lim_ptoinfty|a_2p+1 - a_2p| = 0
                  $$



                  Finally:
                  $$
                  exists lim_ntoinftyS_n implies exists lim_ntoinftysum_k=1^n b_k = L
                  $$

                  Thus:
                  $$
                  lim_ntoinftysum_k=1^n b_k = lim_ntoinfty(a_n - a_1) = L \
                  implies lim_ntoinftya_n = L + a_1
                  $$



                  Thus $a_n$ is convergent.






                  share|cite|improve this answer









                  $endgroup$



                  Ok, following @Kavi Rama Murthy's suggestions here is what I eventually got (hopefully correct).



                  I'm still going to stick to how $b_n$ is defined:
                  $$
                  lim_ntoinftyb_n = 0\
                  |b_n+2| le |b_n+1|\
                  b_n+2b_n+1 le 0
                  $$

                  Let's now define a partial sum $S_n$:
                  $$
                  S_n = sum_k=1^n (-1)^k-1|b_k|
                  $$

                  Here the $(-1)^k-1$ actually depends on which terms are positive and which are negative (which is defined by eiher $(5)$ or $(6)$ in the OP). Consider two partial sums for odd and even numbers of terms:
                  $$
                  beginalign*
                  S_2p+1 = sum_k=1^2p+1(-1)^k-1|b_k| \
                  S_2p = sum_k=1^2p(-1)^k-1|b_k|
                  endalign*
                  $$



                  Consider the sum of odd amount of terms:
                  $$
                  beginalign
                  S_2(p+1)+1 = &sum_k=1^2p+3(-1)^k-1|b_k| \
                  &=sum_k=1^2p+1(-1)^k-1|b_k| underbrace_le0\
                  &le S_2p+1
                  endalign
                  $$

                  Which means $S_2p+1$ is monotonically decreasing.



                  Now use the same approach for even number of terms:
                  $$
                  beginalign
                  S_2(p+1) = &sum_k=1^2p+2(-1)^k-1|b_k| \
                  &=sum_k=1^2p(-1)^k-1|b_k| + underbraceb_2p+2_ge0\
                  &ge S_2p
                  endalign
                  $$

                  Which means $S_2p$ is monotonically increasing. Consider the following:
                  $$
                  S_2p+1 - S_2p = |b_2p+1| ge 0tag*
                  $$



                  Now $S_2 = |b_1| - |b_2|$ and by monotoninity of $S_2p$:
                  $$
                  S_2 = |b_1| - |b_2| le S_2p
                  $$



                  At the same time $S_1 = |b_1|$ which again by monotonicity of $S_2p+1$:
                  $$
                  S_2p+1 le S_1 = |b_1|
                  $$

                  Now using (*):
                  $$
                  |b_1| - |b_2| le S_2p le S_2p+1 le |b_1|\
                  S_2p-1 - S_2p = |b_2p+1| = |a_2p+1 - a_2p|
                  $$

                  But $|a_2p+1 - a_2p|$ is convergent as far as $(a_2p+1 - a_2p)$ is. Which yields:
                  $$
                  lim_ptoinfty(S_2p+1 - S_2p) = lim_ptoinfty|b_2p+1| = lim_ptoinfty|a_2p+1 - a_2p| = 0
                  $$



                  Finally:
                  $$
                  exists lim_ntoinftyS_n implies exists lim_ntoinftysum_k=1^n b_k = L
                  $$

                  Thus:
                  $$
                  lim_ntoinftysum_k=1^n b_k = lim_ntoinfty(a_n - a_1) = L \
                  implies lim_ntoinftya_n = L + a_1
                  $$



                  Thus $a_n$ is convergent.







                  share|cite|improve this answer












                  share|cite|improve this answer



                  share|cite|improve this answer










                  answered Mar 26 at 11:23









                  romanroman

                  2,50221226




                  2,50221226



























                      draft saved

                      draft discarded
















































                      Thanks for contributing an answer to Mathematics Stack Exchange!


                      • Please be sure to answer the question. Provide details and share your research!

                      But avoid


                      • Asking for help, clarification, or responding to other answers.

                      • Making statements based on opinion; back them up with references or personal experience.

                      Use MathJax to format equations. MathJax reference.


                      To learn more, see our tips on writing great answers.




                      draft saved


                      draft discarded














                      StackExchange.ready(
                      function ()
                      StackExchange.openid.initPostLogin('.new-post-login', 'https%3a%2f%2fmath.stackexchange.com%2fquestions%2f3162857%2fa-sequence-a-n-is-such-that-lim-na-n1-a-n-0-given-some-additi%23new-answer', 'question_page');

                      );

                      Post as a guest















                      Required, but never shown





















































                      Required, but never shown














                      Required, but never shown












                      Required, but never shown







                      Required, but never shown

































                      Required, but never shown














                      Required, but never shown












                      Required, but never shown







                      Required, but never shown







                      Popular posts from this blog

                      Lowndes Grove History Architecture References Navigation menu32°48′6″N 79°57′58″W / 32.80167°N 79.96611°W / 32.80167; -79.9661132°48′6″N 79°57′58″W / 32.80167°N 79.96611°W / 32.80167; -79.9661178002500"National Register Information System"Historic houses of South Carolina"Lowndes Grove""+32° 48' 6.00", −79° 57' 58.00""Lowndes Grove, Charleston County (260 St. Margaret St., Charleston)""Lowndes Grove"The Charleston ExpositionIt Happened in South Carolina"Lowndes Grove (House), Saint Margaret Street & Sixth Avenue, Charleston, Charleston County, SC(Photographs)"Plantations of the Carolina Low Countrye

                      random experiment with two different functions on unit interval Announcing the arrival of Valued Associate #679: Cesar Manara Planned maintenance scheduled April 23, 2019 at 00:00UTC (8:00pm US/Eastern)Random variable and probability space notionsRandom Walk with EdgesFinding functions where the increase over a random interval is Poisson distributedNumber of days until dayCan an observed event in fact be of zero probability?Unit random processmodels of coins and uniform distributionHow to get the number of successes given $n$ trials , probability $P$ and a random variable $X$Absorbing Markov chain in a computer. Is “almost every” turned into always convergence in computer executions?Stopped random walk is not uniformly integrable

                      How should I support this large drywall patch? Planned maintenance scheduled April 23, 2019 at 00:00UTC (8:00pm US/Eastern) Announcing the arrival of Valued Associate #679: Cesar Manara Unicorn Meta Zoo #1: Why another podcast?How do I cover large gaps in drywall?How do I keep drywall around a patch from crumbling?Can I glue a second layer of drywall?How to patch long strip on drywall?Large drywall patch: how to avoid bulging seams?Drywall Mesh Patch vs. Bulge? To remove or not to remove?How to fix this drywall job?Prep drywall before backsplashWhat's the best way to fix this horrible drywall patch job?Drywall patching using 3M Patch Plus Primer